6th Junior Balkan Mathematical Olympiad Problems 2002



1.  The triangle ABC has CA = CB. P is a point on the circumcircle between A and B (and on the opposite side of the line AB to C). D is the foot of the perpendicular from C to PB. Show that PA + PB = 2·PD.
2.  The circles center O1 and O2 meet at A and B with the centers on opposite sides of AB. The lines BO1 and BO2 meet their respective circles again at B1 and B2. M is the midpoint of B1B2. M1, M2 are points on the circles center O1 and O2 respectively such that angle AO1M1 = angle AO2M2, and B1 lies on the minor arc AM1 and B lies on the minor arc AM2. Show that angle MM1B = angle MM2B.


3.  Find all positive integers which have exactly 16 positive divisors 1 = d1 < d2 < ... < d16 such that the divisor dk, where k = d5, equals (d2 + d4) d6.
4.  Show that 2/( b(a+b) ) + 2/( c(b+c) ) + 2/( a(c+a) ) ≥ 27/(a+b+c)2 for positive reals a, b, c. 

Solutions
Problem 1
The triangle ABC has CA = CB. P is a point on the circumcircle between A and B (and on the opposite side of the line AB to C). D is the foot of the perpendicular from C to PB. Show that PA + PB = 2·PD.
Solution
Take Q on the ray PB so that PD = DQ. Then CA = CB, CP = CQ, ∠CAP = ∠CBQ, so triangles CAP and CBQ are congruent, so AP = BQ. Hence 2·PD = PQ = PB + PA.
Thanks to Cristian Ilac

Problem 3
Find all positive integers which have exactly 16 positive divisors 1 = d1 < d2 < ... < d16 such that the divisor dk, where k = d5, equals (d2 + d4) d6.
Answer
2002
Solution
If d2 > 2, then all divisors are odd. But (d2 + d4) is a divisor and must be even. Contradiction. So d2 = 2.
If d3 = 3, then 6 divides N (where we write the number as N), so either d4 = 6, or d5 = 6 or d6 = 6. If d4 = 6, then (d2 + d4) = 8 is a divisor, so 4 is a divisor, so d4 = 4. Contradiction. If d5 = 6, then we are given that d6 = (d2 + d4) d6 > d6. Contradiction. If d6 = 6, then d4 = 4, d5 = 5. But we are given that d5 = (d2 + d4) d6 > d5. Contradiction. So d3 > 3. So 3 is not a factor of N.
We are told that (d2 + d4) d6 is a factor, so (d2 + d4) = d4 + 2 must be a factor. If d4 + 1 is also a factor, then we have three consecutive factors. One of them must be a multiple of 3. But we have just shown that 3 does not divide N. So d4 + 1 is not a factor. Hence d5 = d4 + 2. Neither d4 nor d5 are multiples of 3. So d5 must be 1 greater than a multiple of 3.
We have d5 = k and that dk is a factor. So in particular k ≤ 16 (since there are only 16 factors). Thus d5 = 1, 4, 7, 10, 13 or 16. Obviously d5 ≥ 5, so 1 and 4 are too small.
If d5 = 7, then since 3 and 6 are not factors we must have d3 = 4, d4 = 5. So 10 and 14 are factors. So d7 ≤ 10. But we are given that d7 = (d2 + d4) d6 > (2 + 5) 7 = 49. Contradiction.
If d5 = 10, then d4 = 8. So 4 and 5 are also factors, but only d3 is available. Contradiction.
If d5 = 16, then d4 = 14. So 4, 7 and 8 are also factors, but only d3 is available. Contradiction.
The only remaining possibility is d5 = 13. That implies d4 = 11. So d3 is the only factor between 4 and 10. So it cannot be 5, 6, 8, 9 or 10. So it must be 4 or 7.
Suppose it is 4. N has prime factors 2, 11, 13. If it has another prime factor p, then it would have 16 factors, leaving aside multiples of 4 (because each of 2, 11, 13 and p could be in or out). Contradiction. So 2, 11, 13 are its only prime factors. If 112 or 132 divides N, then N has at least 3·3·2 = 18 factors (there are 3 possibilities for 2, 3 for one of 11/13 and 2 for the other). Contradiction. So 23must divide N. Contradiction (we are supposed to have exhausted the factors under 10).
So the only remaining possibility is d3 = 7. That gives N = 2002 and it is easy to check that d13 = 182 = (d2 + d4) d6.
Comment. This all seems rather complicated (although it did not take me long). Does anyone have a simpler solution?

Problem 4
Show that 2/( b(a+b) ) + 2/( c(b+c) ) + 2/( a(c+a) ) ≥ 27/(a+b+c)2 for positive reals a, b, c.
Solution
Thanks to Michael Lipnowski
By AM/GM 1/( b(a+b) ) + 1/( c(b+c) ) + 1/( a(c+a) ) ≥ 3/(XY), where X = (abc)1/3, Y = ((a+b)(b+c)(c+a))1/3. Using AM/GM again gives X ≤ (a+b+c)/3 and Y ≤ 2(a+b+c)/3, so 3/XY ≥ (27/2) 1/(a+b+c)2.


Fun Maths Games for Kids

 
Return to top of page Copyright © Math Learning - Yearbooks - School Books - School Reading Books - Learning Math for Kids - Kids Math Learning - Math Games for Kids - Math Books for Kids - Online Math learning - Maths Learning - Online Math Learning - Math learning software - Math Learn - Math Learning Disabilities - Math Playground - Math is Fun - Math Learning center - Math Online - 3 digit divisor worksheets - Math Olympiad - Math Games Olympiad 2010 www.mathlearning.org. All right reseved. | Powered by Kids Math Books